Fix V_B typos in Serway and Jewett v8's 25.16 solution.
[course.git] / latex / problems / Serway_and_Jewett_8 / problem25.16.tex
index f6e2fa786132801790f32c0aba4218e04666ac72..c20016d044194dcb403902df1f2d9ee888bcc114 100644 (file)
@@ -35,9 +35,9 @@ $2Q$ charge to $A$ ($\sqrt{d^2 + d^2} = \sqrt{2d^2} = d\sqrt{2}$).
 \Part{b}
 The geometry is flipped for $B$, so we have
 \begin{equation}
-  V_A = k\frac{Q}{d\sqrt{2}} + k\frac{2Q}{d}
+  V_B = k\frac{Q}{d\sqrt{2}} + k\frac{2Q}{d}
     = k\frac{Q}{d}\p({2+\frac{1}{\sqrt{2}}})
-    = \ans{6.08\E{-15}\U{V}} = \ans{6.08\U{kV}} \;,
+    = \ans{6.08\E{3}\U{V}} = \ans{6.08\U{kV}} \;,
 \end{equation}
 
 \Part{c}